2
$\begingroup$

A $(n,n/2,\lambda)$ block-design is a family $A_1,...,A_K$ of subsets of $[n]$ such that $|A_i|=n/2$ and for every $1 \leq i < j \leq n$ it holds that $\#\{1 \leq k \leq K : i,j \in A_k \} = \lambda$. My question is: What is the minimal $K$ for which such a design exists?

$\endgroup$
2
  • $\begingroup$ If you fix both $n$ and $\lambda$, this determines $K$ if the design exists. If you fix $n$ and let $\lambda$ vary, this requires you to be able to determine whether designs with particular parameters exist, which is hard in general. $\endgroup$ Apr 2, 2016 at 18:39
  • $\begingroup$ To clarify Douglas's comment, K will be $\lambda(2n-2)/(n/2-1)$. However, even if K is a positive integer, that is not enough to guarantee the existence of the design with the desired parameters. Gerhard "Has Designs On Some Designs" Paseman, 2016.04.02. $\endgroup$ Apr 2, 2016 at 19:24

2 Answers 2

2
$\begingroup$

You can take all of the $2$-subsets of $[4]$ to get a $(4,2,1)$ design with $6$ blocks. This is the smallest possible number of blocks, but maybe you would want to excuse it for being trivial. There is also an $(8,4,3)$ design with $14$ blocks. I'm not sure if there is anything in between.

$\endgroup$
1
$\begingroup$

$K=10$. The ten triangles in the image below form the blocks of a $2\text{-}(6,3,2)$ design. rp2

Image from here: https://math.stackexchange.com/a/366455/65

This is optimal if you insist that your design is not $\binom{[n]}{n/2}$.

We have $nr=Kn/2$ where $r$ is the replication number. So K is even.
For $K=8,4,2$, there are no solutions $(n,\lambda)\in \mathbb N^2$ to $K=\frac{2\lambda (n-1)}{n/2-1}$. For $K=6$, you have $n=4$, which gives the trivial case.

Or you could look up the Design Database.

$\endgroup$

Your Answer

By clicking “Post Your Answer”, you agree to our terms of service and acknowledge you have read our privacy policy.

Not the answer you're looking for? Browse other questions tagged or ask your own question.